Renal System/Urology/Male Reproductive System Flashcards

1
Q

A 35-year-old man with history of sore throat three weeks ago, presents to the Emergency Department with a puffy face and decreased urine output for the past 48 hours. On examination, he has a blood pressure of 160/120 mmHg. His face is swollen especially in the periorbital area. On lung auscultation, bilbasal crepitations are heard. Which one of the following is correct regarding his condition?

A. Hematuria is a grave prognostic factor.
B. He is at immediate risk of death from left ventricular failure.
C. Increase in oral intake will result in diuresis.
D. If there is renal tenderness, a renal biopsy should be performed.
E. Dialysis is contraindicated during the acute phase of the illness.

A

Correct Answer Is B.

Facial edema, hypertension and oliguria are strong pointers towards glomerulonephritis. With the sore throat in the history, post-streptococcal glomerulonephritis (PSGN) is the most likely cause of this presentation.

PSGN is induced by infection with specific nephritogenic strains of group A streptococcus (GAS) such as type 12 and type 49. The clinical presentation can vary from asymptomatic, microscopic hematuria to full-blown acute nephritic syndrome, characterized by red to brown urine, proteinuria (which can reach the nephrotic range), edema, hypertension and acute renal failure.
A latent period always occurs between the streptococcal infection and the onset of signs and symptoms of acute glomerulonephritis. In general, the latent period is 1-2 weeks after a throat infection and 3-6 weeks after a skin infection. The onset of signs and symptoms at the same time as pharyngitis is points towards immunoglobulin A (IgA) nephropathy rather than PSGN.

Dark urine (brown-, tea-, or cola-colored) is often the first clinical manifestation of PSGN. Dark urine is caused by lysis of red blood cells that have penetrated the glomerular basement membrane and have passed into the tubular system.

Periorbital edema is typical. The onset of puffiness of the face or eyelids is sudden. It is usually prominent upon waking up and, if the patient is active, tends to subside toward the end of the day.

In some cases, generalized edema and other features of circulatory congestion, such as dyspnea, may be present. Edema is the result of a defect in renal excretion of salt and water. The severity of edema is often disproportionate to the degree of renal impairment. Nonspecific symptoms of PSGN can include general malaise, weakness, and anorexia that are present in 50% of patients.

Approximately 15% of patients complain of nausea and vomiting.

Early death is extremely rare in children (<1%) but is significantly more common in adults (25%).

This is secondary to congestive heart failure and azotemia. Left ventricular failure (congestive heart failure) is more common in adults (43%) than in children (<5%); therefore, this patient is potentially at immediate risk of congestive heart failure and death due to volume overload.

Nephrotic-range proteinuria is also more common in adults (20%) than in children (4-10%). Approximately 83% of adults have azotemia compared to 24-40% of children.

(Option A) Hematuria is seen in most patients with PSGN as in all other forms of glomerulonephritis. It can range from microscopic to gross (cola-or tea-colored urine). Hematuria is not associated with poor prognosis.

(Option C) Oliguria is present in 10-50% of cases. The oliguria is often transient and diuresis occurs within 1-2 weeks. Increased water intake does not result in diuresis.

(Option D) Kidney biopsy is not required, unless for those patient in whom other glomerular disorders are considered due to deviation from the natural course of PSGN or lack of a history of streptococcal infection. Recognition of PSGN in this patient is clear from the history and the clinical findings. Renal tenderness is not an indication for renal biopsy.

(Option E) Patients with PSGN have variable reduction in renal function, and some patients require dialysis during the acute episode.

How well did you know this?
1
Not at all
2
3
4
5
Perfectly
2
Q

A 72-year-old man presents with complaint of one episode of blood in the urine. Five years ago, he underwent colectomy after he was diagnosed with colon cancer. A while back, he developed back pain for which he was assessed and diagnosed with metastatic bone disease. Three months ago he was started on tramadol for management of the back pain. He has no urinary symptoms. Which one of the following is the most appropriate next best step in management?

A. Perform a pelvic and abdominal CT scan.
B. Perform a renal ultrasound.
C. Urine culture.
D. Stop tramadol.
E. Perform an intravenous pyelogram (IVP).

A

Correct Answer Is C.

Hematuria should be always considered as a sign; therefore, for established cases of hematuria an underlying cause should be investigated through history, physical examinations and laboratory/imaging studies. For established hematuria, the most common causes such as vigorous exercise, menstruation, trauma, viral illnesses, and infections should be excluded first.

One of the most common causes of hematuria even in the absence of symptoms is urinary tract infection (UTI), for which a urine analysis and culture should be performed.

(Option A) Genitourinary malignancies (either primary or metastatic) can cause hematuria. Malignancies should always be excluded and CT scan is one of the most accurate and commonly used means. However, other common causes of hematuria should be excluded first.

(Option B) Renal ultrasound may be later needed as further work-up but not as the first priority because the hematuria has been painless, making renal stones less likely. Small stones may cause painless hematuria, but since they almost always pass spontaneously, their detection on sonography does not change the management plan.

(Option D) Although hematuria has been reported as a rather rare adverse effect of tramadol, initial assessment should be focused on more common possibilities such as UTI. Even so, cessation of tramadol in an almost end-stage patient would not be recommended.

(Option E) If imaging studies are required during the evaluation process, CT scan is preferred over IVP. In the presence of more modern and convenient modalities, IVP is rarely done these days.

How well did you know this?
1
Not at all
2
3
4
5
Perfectly
3
Q

A 65-year-old man presents to your practice with complaint of blood in the urine. These episodes of hematuria have all been painless. Which one of the following is the least likely cause of painless hematuria?

A. Cancer within the kidney.
B. Anticoagulation therapy.
C. Glomerulonephritis.
D. Benign prostatic hyperplasia (BPH).
E. Use of cyclophosphamide.

A

Correct Answer Is E.

All the given options can present with painless hematuria except cyclophosphamide.

Cyclophosphamide use can result in hemorrhagic cystitis. With cystitis, the hematuria is more likely to be painful rather than painless.

The following can result in hematuria (painless or painful):
* Urothelial cancers (kidney cancers, bladder cancers, etc)
* Hydronephrosis/distention
* Renal vein thrombus / renal artery embolism
* Arteriorvenous malformation
* Papillary necrosis (sickle cell disease)
* Hypertension
* Glomerulonephritis
* Structural abnormalities (polycystic kidney disease, medullary sponge kidney, etc)
* Nephrolithiasis
* Urinary tract infections (pyelonephritis, cystitis, parasitic infections, etc)
* Ureteral strictures
* BPH
* Prostate cancer
* Prostatic procedures
* Trauma (including traumatic catheterization)
* Exercise-induced hematuria
* Bleeding diathesis / anticoagulation
* Urethritis
* Urethral diverticulum
* Hypercalciuria/hyperuricosuria
* Urinary tract fistulas
* Mimics of hematuria:
* Menstruation
* Drugs (phenazopyridine, pyriduim, rifampin, nitrofurantoin, etc)
* Pigmenturia
* Beeturia

How well did you know this?
1
Not at all
2
3
4
5
Perfectly
4
Q

A couple has presented to your office for infertility consult. They have been trying to start a family for over a year. The woman’s history and examination results are normal, but the man is found to have absent vas deferens. Which one of the following would be the most appropriate advice for
them?

A. They should have sperm donation.
B. The vas deferens should be fused to the ejaculatory duct.
C. Sperm aspiration and intrauterine fertilization should be tried.
D. Sperm aspiration for frozen sample and repeated tubal insemination.
E. In vitro fertilization.

A

Correct Answer Is E.

One to two percent of infertile men have congenital absence of the vas deferens. Most have mutations of the cystic fibrosis transmembrane conductance regulator (CFTR) gene. Many infertile men with mutations of CFTR present with infertility in the absence of other typical features of cystic fibrosis (e.g. respiratory and pancreatic disease).

Using in vitro fertilisation is the most appropriate reproductive technique for this couple. Sperms are aspirated from epididymis or testis and then injected into the aspirated oocytes in vitro.

NOTE - Patients with congenital bilateral absence of the vas deferens may have genetic mutation commonly present in cystic fibrosis. Such men and their partners who are considering assisted reproductive techniques to achieve pregnancy should have genetic screening and counselling.

Screening the female partner may be more cost effective than screening the patient because if she
is negative, the risk that their children having cystic fibrosis is almost zero.

(Option A) Sperm donation, while the male sperms are retrievable, is not an appropriate treatment option.

(Option B) Fusion of vas deferens to ejaculatory duct is considered appropriate for male patients,
who have vas deferens but has undergone vasectomy.

(Options C and D) Aspirated sperms are not enough in number to make intrauterine infertilization
or tubal insemination acceptable methods.

How well did you know this?
1
Not at all
2
3
4
5
Perfectly
5
Q

A 65-year-old man, who is on chronic hemodialysis every other day due to chronic renal failure, presents to the Emergency Department with generalized weakness and dyspnea. He is noncompliant with his dialysis sessions and had his last session 5 days ago. Which one of the following would be the most appropriate **next action **to take?

A. Call the dialysis unit.
B. Arterial blood gas (ABG) analysis.
C. ECG.
D. Chest X-ray (CXR).
E. Furosemide.

A

Correct Answer Is A.

This patient has skipped dialysis sessions. Patients with end stage renal disease have volume overload and hazardous metabolic derangements such as metabolic acidosis and hyperkalemia, both of which can be fatal. Volume overload can cause congestive heart failure and pulmonary edema presenting with dyspnea as the most common manifestation. Hyperkalemia is also of great concern because it can cause lethal cardiac arrhythmias. No matter what the underlying cause of this presentation is, arrangement for urgent dialysis is the most appropriate next step in management as it shoud be arranged at earliest time. While the arrangements are in process,
investigations such as ECG (option C), CXR (option D) and ABG (option B) can be undertaken.

ECG is of paramount importance because with evidence of arrhythmias induced by hyperkalemia,
calcium gluconate should be administered promptly for cardiac protection.

Urgent dialysis can remove excess fluid and the congestion; in the mean time, standard protocols
should be used for such patients. In case of pulmonary edema, furosemide (option E) can be used in patients with preserved renal function as a component of acute management of pulmonary
edema.

How well did you know this?
1
Not at all
2
3
4
5
Perfectly
6
Q

A 70-year-old woman presents with increasing generalized abdominal pain for the past 2 hours. She is a known case of chronic renal failure and has been on peritoneal dialysis for the past 18 months. Her last dialysis session was hours ago. On examination, she appears toxic. Her blood pressure is 140/95mmHg, heart rate 110bpm, and temperature 39°C. Her abdomen is distended and diffusely tender to deep palpation. Rebound tenderness is also noted. Blood exam is significant for a white cell count of 18,000/mm3. Which one of the following is the most appropriate next step in management?

A. Erect and supine abdominal films.
B. Ultrasonography of the abdomen.
C. CT scan of the abdomen and pelvis.
D. Peritoneal fluid analysis and Gram stain.
E. Comparison of ascitic fluid amylase with serum amylase.

A

Correct Answer Is D.

In the setting of peritoneal dialysis (PD), **peritonitis ** is the most likely explanation to this presentation.

Peritonitis is a common complication of peritoneal dialysis (PD), and is associated with significant morbidity, catheter loss, transfer to hemodialysis, transient loss of ultrafiltration, possible permanent membrane damage, and occasionally death.

Among PD patients, peritonitis may be PD-related or secondary (enteric). PD-related peritonitis is due to touch contamination with pathogenic skin bacteria or to catheter-related infection.

Secondary peritonitis is caused by underlying pathology of the gastrointestinal tract. PD-related peritonitis is much more common than secondary peritonitis; the latter, however, is associated with higher rates of complications and mortality.

Conditions that may lead to secondary peritonitis include cholecystitis, appendicitis, ruptured diverticulum, treatment of severe constipation, perforation during endoscopy, bowel ischemia, and incarcerated hernia. Secondary peritonitis may also be caused by seeding from the blood or vagina but this is less common compared to intraabdominal causes.

The most common signs and symptoms of peritonitis among PD patients are abdominal pain and cloudy peritoneal effluent. Other signs and symptoms include fever, nausea, diarrhea, abdominal tenderness, rebound tenderness, and occasionally systemic signs, including hypotension.

The diagnosis of peritonitis should be suspected in a PD patient with characteristic clinical presentation. Analysis of the peritoneal fluid is the most appropriate next step when PD-related peritonitis is suspects.

A presumptive diagnosis is made if the peritoneal fluid white cell count is greater than 100cells/mm3 and the percentage of neutrophils is greater than 50%. The white cell count is greatly dependent on the dwell time of the dialysis catheter; therefore, with a white cell count of less than 100, but neutrophil percentage of greater than 50%, the diagnosis remains PD-related peritonitis untill proven otherwise.

Peritoneal fluid culture and Gram stain should also be performed. Although it is usually negative, the identification of any organisms is a helpful guide to therapy since the gram stain is predictive of the culture results. Gram stain may be particularly useful in the early diagnosis of fungal peritonitis.

Peritoneal fluid cultures are always indicated and performed, but empiric antibiotic therapy should not be delayed until culture results are available.

Culture of purulent drainage from the exit site should be performed since isolation of the same organism as from peritoneal fluid suggests that the exit site infection may be the cause of peritonitis.

Peritoneal fluid amylase and lipase should also be measured. The peritoneal fluid amylase and lipase concentrations are occasionally elevated (>50IU/L) among patients with secondary peritonitis, but not among patients with PD-related peritonitis. This may help to distinguish between PD-related and secondary peritonitis. Among patients with secondary peritonitis, the elevated amylase concentration may be due to a leak from a bowel perforation or directly from an inflamed pancreas. An elevated level of peritoneal lipase above 15IU/L suggests pancreatitis as the cause of the peritonitis.

NOTE – PD-related peritonitis is a different entity from spontaneous bacterial peritonitis (SBP).

(Options A, B and C) Radiographic manifestations of peritonitis are nonspecific and such studies are not routinely performed among patients with suspected PD-related peritonitis. However, they are indicated when secondary peritonitis is suspected.

(Option E) Amylase and lipase levels in peritoneal fluid are not used in comparison with their serum levels.

In the presence of systemic symptoms, blood cultures should be obtained, although they are seldom positive.

How well did you know this?
1
Not at all
2
3
4
5
Perfectly
7
Q

A 60-year-old man with end-stage renal failure presents to the dialysis unit for a session of hemodialysis. His current medications are enalapril, hydrochlorothiazide, and aspirin. He has a blood pressure of 140/95 mmHg. After the session, which was uneventful, his blood pressure is found to be 160/90mmHg. Which one of the following is more likely to be the cause of his increased blood pressure?

A. Allergic reaction to the dialysis fluid.
B. Overdialysis.
C. Enalapril overdose.
D. Hypokalemia.
E. Anemia.

A

Correct Answer Is B. Overdialysis.

While hemodialysis lowers blood pressure (BP) in most hypertensive end-stage renal disease patients, approximately 15% of patients show a paradoxical increase in BP during haemodialysis, termed intradialytic hypertension (IDH). IDH is defined as either of the following:
* An increase in mean arterial blood pressure (MAP) ≥ 15 mmHg during or immediately after hemodialysis
* An increase in systolic BP (SBP) >10 mmHg from pre- to post-dialysis
* Hypertension during the second or third hour of hemodialysis after significant ultrafiltration has taken place
* An increase in BP that is resistant to ultrafiltration
* Aggravation of pre-existing hypertension or development of de novo hypertension with erythropoietin stimulating agents.

Despite extensive investigations as to the mechanism and pathophysiology of IDH, the exact pathogenesis remains unclear. Numerous factors have been implicated, including:
1. Renin-angiotensin system activation because of ultrafiltration (UF) induced hypovolemia
2. Sympathetic overactivity
3. Intradialytic Ca++/k+ variations
4. Blood viscosity/haemoconcentration-induced vasoconstriction caused by erythropoietin treatment
5. Fluid overload
6. Increased cardiac output
7. Endothelin-driven vasoconstriction
8. Antihypertensive drug removal by dialysis treatment

Several studies and reports highlight the important role of fluid overload, hemodynamic changes and increased endothelin level. The importance of other hypothesis such as renin-angiotensin system activation, sympathetic overactivity and ionic variations seems secondary. Fluid removal remains the key point for treatment of IDH.

Of the options, overdialysis is the most appropriate one. In majority of dialysis patient, overdialysis results in hypotension as a complication; in some, however, this excess fluid removal leads to activation of compensatory mechanisms by which the blood pressure can increase. Also, excess removal of enalapril with dialysis can be an explanation.

(Option A) Allergic reaction to dialysis fluid has not been shown as a possible mechanism responsible for intradialytic hypertension.

(Option C) Enalapril overdose can cause hypotension, not hypertension.

(Option D) Hypokalemia induces vasoconstriction and increased blood pressure; however, it is does not appear to be a main contributor to IDH.

(Option E) Anemia, by itself, is not associated with hypertension during or shortly after dialysis, but erythropoietin stimulating agents used for treatment of anemia in patients with CKD during dialysis is among postulated causes of hypertension associated with dialysis.

How well did you know this?
1
Not at all
2
3
4
5
Perfectly
8
Q

A 64-year-old man with end-stage renal disease presents to the dialysis unit for a pre-scheduled
hemodialysis session. He proceeds through the session uneventfully, but his blood pressure reading is 80/60 mmHg at the end of the session. His pre-dialysis blood pressure was 150/90mmHg. A blood panel shows a hemoglobin level of 80 g/L. Which one of the following could be the **most likely cause **of this drop in blood pressure?

A. Allergic reaction to the dialysis fluid.
B. Overdialysis.
C. Anemia.
D. Hypokalemia.
E. Enalapril overdose.

A

Correct Answer Is B.

The scenario is a typical example of intradialytic hypotension. Intra- or post-dialysis hypotension is
a well recognized and common complication of hemodialysis.

The incidence of a symptomatic reduction in blood pressure during (or immediately following) dialysis ranges from 15-50% of dialysis sessions. In some patients, the development of orthostatic
hypotension necessitates intravenous fluid replacement before they are able to leave the dialysis
unit. This problem contributes to the excessive morbidity associated with the dialysis procedure.
There are two clinical patterns of dialysis-associated hypotension:
1. Episodic hypotension, which typically occurs during the later stages of dialysis and presents with vomiting, muscle cramps, and other vagal symptoms (such as yawning).
2. Chronic persistent hypotension, which may occur in patients with long-term dialysis. A predialysis systolic blood pressures of less than 100 mmHg is frequently observed.

The etiology of intradialytic hypotension is divers and includes:
* A rapid reduction in plasma osmolality, which causes extracellular water to move into the cells
* Rapid fluid removal in an attempt to attain “dry weight”, particularly among those with large inter-dialytic weight gains
* Inaccurate determination of true ‘dry weight’
* Autonomic neuropathy
* Diminished cardiac reserve
* Use of acetate rather than bicarbonate as a dialysate buffer
* Intake of antihypertensive medications that can impair cardiovascular stability
* Use of a lower sodium concentration in the dialysate
* Sudden release of adenosine during organ ischemia
* Ingestion of a meal immediately before or during dialysis
* Arrhythmias or pericardial effusion with tamponade, which are volume-unresponsive
* Reactions to the dialyzer membrane, which may cause wheezing and dyspnea as well as hypotension
* Increased synthesis of endogenous vasodilators, such as nitric oxide
* High magnesium concentrations in the dialysate
* Failure to increase plasma vasopressin levels

Overdialysis, by excess fluid removal, is an important, and probably the most common cause of intradialytic hypotension.

(Option A) Allergic reaction to dialyzer membrane (not fluid) can be a possible cause of hypotension in few cases. In addition to hypotension, wheezing and dyspnoea are also expected.

(Option C) Anemia has not been recognized as a cause of intradialytic hypotension.

(Option D) Hypokalemia induces vasoconstriction, with increased blood pressure being the expected result. Hypokalemia is unlikely to cause hypotension in this setting.

(Option E) Enalapril overdose can cause hypotension regardless of dialysis. Circulatiing enalapril is expected to decrease during dialysis; therefore, enalapril overdose, while the patient has not have hypotension before the session is very unlikely.

How well did you know this?
1
Not at all
2
3
4
5
Perfectly
9
Q

A 72-year-old woman has been on routine dialysis sessions for the past 6 months due to end-stage renal disease. At the beginning of each session, she is found to have a high blood pressure (BP).

During the session the BP normalizes, but goes up again after the session. Which one of the following would be the most appropriate management option for this patient?

A. Addition of hypertensive medications.
B. Addition of furosemide, daily.
C. Increasing the dialysis time.
D. Sedation before dialysis.
E. Decreasing the dialysis time.

A

Correct Answer Is C.

Although hypotension during hemodialysis is a frequent complication, some patients (5-15%) develop paradoxical hypertension in the later stages of dialysis or when the patients comes off the dialysis machine, a time at which most of the excess fluid has already been removed. The pathogenesis is unclear; however, the following mechanisms have been hypothesized as the cause:
1. Renin-angiotensin system activation because of ultrafiltration (UF) induced hypovolemia
2. Sympathetic overactivity
3. Intradialytic Ca++/k+ variations
4. Blood viscosity/hemoconcentration-induced vasoconstriction caused by erythropoietin treatment
5. Fluid overload
6. Increased cardiac output
7. Endothelin-driven vasoconstriction
8. Antihypertensive drug removal by dialysis treatment

The optimal therapy for this problem is not known.

While antihypertensive medications such as angiotensin converting enzyme inhibitors (ACE inhibitors) and alpha-blockers have been used before (or during) dialysis, they have not been predictably effective. Carvedilol, which blocks endothelin-1 release, appears to be effective.

Although there are no validated universal guidelines regarding management of such patients, fluid removal has been accepted as the first-line treatment for intradialytic hypertension (IDH).

Theoretically, increasing the time of the dialysis session and ultrafiltration (UF) rate would be efficient; however, this decision faces many difficulties such as patient refusal or the unit limitations. This treatment should be done with caution to avoid hazardous blood pressure drop that may occur in the elderly or patients with severe comorbidity.

The dry weight of patients should be gradually reduced by increasing the dialysis time and the UF rate. In addition, patients should be advised to decrease their daily salt and water intake in between their dialysis sessions.

(Options A and B) Addition of anti-hypertensive or other medications ACE inhibitors or angiotensin receptor blockers (ARBs), beta blockers, endothelin-1 receptor blockers, furosemide, etc has been associated with conflicting results. In some patients, hypertensive crises may occur. These hypertensive crises are not persistent and usually the blood pressure level quickly decreases spontaneously; however, addition of antihypertensive medications might be indicated. There is no comment regarding a hypertensive crisis in this patient to necessitate addition of antihypertensives. Furosemide is not the first-line option for management of hypertension crisis if it occurs.

(Option D) Sedation before dialysis has no role in management of IDH or post- dialytic hypertension.

(Option E) Decreasing the dialysis time results in insufficient excess fluid removal and hypertension due to volume overload.

How well did you know this?
1
Not at all
2
3
4
5
Perfectly
10
Q

A 50-year-old man presents with complaints of several episodes of painless gross hematuria and a mass he felt in his left loin recently. The significant finding on examination is a non-tender loin mass. Urine analysis shows blood in the urine. Urine culture is negative. Which one of the following is the most appropriate next step in management?

A. CT scan.
B. Cystoscopy.
C. MRI.
D. Intravenous pyelography (IVP).
E. Intravenous urogram (IVU).

A

Correct Answer Is A.

Gross hematuria is always concerning and warrants thorough investigations, because the prevalence of urinary tract malignancies among patients with macroscopic hematuria has been reported to be as high as 19%, but usually ranges from 3-6%.

Risk factors can help in determining which patients are at higher risk of urinary tract and bladder malignancies. Risk factors include:
* Age >40 years
* A history of smoking
* History of gross hematuria
* History of chronic cystitis or irritative lower urinary tract symptoms (e.g. frequency, urgency,
* dysuria, nocturia, hesitancy, sensation of incomplete emptying)
* History of pelvic irradiation
* Exposure to occupational chemical and dyes (e.g. heavy phenacetin use, treatment with high
* doses of cyclophosphamide, aristolochic acid)

One of the most common causes of hematuria is urinary tract infection, which can be investigated with urine microscopy, culture and sensitivity (MC&S). Ureteric and renal stones are another common cause, but these typically present with pain and microscopic or gross hematuria. In cases where macroscopic hematuria or risk factors are present, or if another cause cannot be determined, more extensive investigations are recommended to exlcude an underlying malignancy.

Despite extensive investigation, studies have shown that in up to 50% of patients with macroscopic hematuria and 70% with microscopic hematuria have no identifiable cause is found. This could be attributed to transient benign physiological conditions, including vigorous physical exercise, sexual intercourse or menstrual contamination.

In this patient, urinalysis is negative but for blood. There is no increase in WBC to indicate infection; furthermore, a urine culture is negative, excluding infections as a cause. There are also no red cell casts or other pathologic findings indicating conditions such as glomerulonephritis as the underlying pathophysiology. An enlarged or otherwise abnormal prostate would have come to attention on physical exam.

Given the presence of gross hematuria, the age (>40) and the loin mass, the most important concern would be a renal tumor until proven otherwise.

Such patients should undergo non-contrast CT scan of kidney, ureter and bladder (KUB) (first-line) or ultrasound (second-line). CT-KUB is a non-contrast study – the current gold standard in identifying ureteric and renal stones, with 94-98% sensitivity; however, the sensitivity is much lower for indentifying genitourinary malignancies.

CT scan findings associated with increased risk of urinary tract malignancies justify CT-KUP IVP. CT-KUB IVP (multidetector CT scanning of kidneys, ureter and bladder after intravenous contrast media is administered), also known as CT urography (CTU) is as accurate and diagnostic as a combination of ultrasonography, IVP and CT-KUB. This, however, should be performed by, or at least under consultation with an urologist. Sensitivity for detecting pathology in patients with hematuria varies from 94-100%, with a 97.4% specificity.

With the loin mass, the cause of hematuria is more likely to originate from upper parts of urinary tract that are beyond the reach of cystoscopy. Cystoscopy, however, can be considered somewhere along the diagnostic pathway, for patients with voiding symptoms or where there is suspicion of bladder cancer, but not as the next best step in this scenario.

IVP (intravenous pyelogram), also called IVU (intravenous urogram) is inferior to CT scan.

How well did you know this?
1
Not at all
2
3
4
5
Perfectly
11
Q

A 32-year-old man presents to the Emergency Department with left loin pain and hematuria. He is HIV positive and on an anti-HIV treatment regimen including indinavir. This is the first time he is having such problem. Urine dipstick is positive for blood but negative for urinary tract infection.

Which one of the following is the most appropriate next step in management?

A. Intravenous pyelogram (IVP).
B. Ultrasound (US).
C. Non-contrast CT scan.
D. KUB X-ray.
E. Triple-phase CT scan.

A

Correct Answer Is B. Ultrasound

Indinavir sulfate is an HIV protease inhibitor. Indinavir-induced nephrolithiasis is a well-recognized adverse effect of this drug, occurring in almost 12.4% of patients. If indinavir-induced nephrolithiasis develops, the drug should be discontinued and an alternative antiretroviral be used.

Non-contrast helical CT scan (option C) is the criterion standard for evaluation of nephrolithiasis. This modality can detect both stones and urinary tract obstruction. Stones not visualized on IVP or KUB, usually are detected on CT scan; however, since stones secondary to indinavir are not radiopaque and signs of obstruction may be minimal, this modality may not be as accurate and miss the diagnosis. This holds true about x-ray KUB (option D) and ultrasound.

Although not accurate as non-contrast helical CT scan, ultrasound can detect obstruction caused by large, clinically significant stones and probably the stone itself. This modality has been recommended as the initial diagnostic option by many authors. It is safe, readily available, costeffective, and spares patients from the risk of radiation in most cases. Again, the accuracy is reduced in detection of stones secondary to indinavir.

In fact, studies have shown that contrast CT-scan is more likely to visualize indinavir-induced stones, but since most of such stones, especially if not large enough to cause obstruction and be detected on ultrasonography, will pass with conservative management, contrast enhanced imaging studies as contrast CT or IVP (option A) unnecessarily puts the patient at risk of radiation and/or contrast media.

Contrast CT scan, however, should be considered for patients in whom the symptoms persist or other diagnosis is suspected based on clinical grounds.

Triple phase CT scan (option E) is mostly used for diagnosis of liver lesions. It is not indicated diagnosis of renal stones but may be considered as an alternative diagnosis.

How well did you know this?
1
Not at all
2
3
4
5
Perfectly
12
Q

During a routine health assessment, a 43-year-old man is found to have established asymptomatic hematuria. Which one of the following is the most appropriate next step in management?

A. Ultrasonography of the renal system.
B. CT scan.
C. Cystoscopy.
D. Urine culture.
E. Intravenous pyelogram (IVP).

A

Correct Answer Is D.
Hematuria may be seen in a variety of situations including, but not limited to:
* Infections
* Renal stones
* Glomerulonephritis
* Urinary tract malignancies
* Trauma

Infections are the most common cause of hematuria in both symptomatic and asymptomatic patients. For this reason, the next best step in management of every patient with hematuria would be a urine analysis and culture. Urologic malignancies (mostly cancer of the bladder) should be considered and ruled out once urine culture is negative and renal stones are excluded using ultrasound (the best initial test) or spiral CT (the most accurate test).

How well did you know this?
1
Not at all
2
3
4
5
Perfectly
13
Q

A 75-year-old man presents to your practice after he noticed blood in his urine. A four-phase contrast CT scan is obtained that shows a 1cm mass in the left kidney, highly suggestive of renal cell carcinoma. His medical history is significant for acute myocardial infarction 1 month ago for which he underwent angioplasty and drug-eluting stent placement. Currently, he is on dual antiplatelet therapy (DAPT) with daily clopidogrel 75mg and aspirin 100mg. Which one of the following would be the most appropriate next step in management ?

A. Nephrectomy.
B. Heminephrectomy.
C. Repeat the CT scan in 12 months.
D. CT-guided percutaneous biopsy.
E. Repeat the CT scan in 3 months.

A

Correct Answer Is C.

Small renal masses can be found incidentally, or during evaluation for urinary symptoms, often by ultrasound. Once a mass is found in the kidney on ultrasound, a four-phase contrasted CT scan should follow for further evaluation, provided renal function allows use of contrast media. These four phases include: (1) arterial, (2) corticomedullary, (3) nephrographic and (4) excretory phases. This study allows a detailed examination of each aspect of the functional anatomy of the kidney.

The majority of renal lesions are benign simple cysts, which would require no further work-up. However, completely solid, mixed (solid and cystic), and cystic renal lesions that enhance with contrast are likely to be malignant.

A small renal mass is defined as a mass < 4 cm that has enhancement on abdominal contrast imaging. For such lesions, surgical resection of the mass is the most appropriate option if life expectancy is >5 years and the patient is a good candidate for surgery.

However, this patient has had an MI one month ago. Major surgery within the first 6 months after MI is associated with high risk of mortality; therefore, this patient is not a candidate for surgical resection of the mass. In such cases active surveillance of a small renal mass with CT or MRI in 6-to 12-month intervals is recommended.

Even if a small renal mass has imaging characteristics highly suspicious for RCC, active surveillance may be appropriate, particularly in patients with medical comorbidities that can increase the risk of active intervention such as surgery, in elderly patients, and in those with decreased life expectancy (≤5 years). Renal impairment may also be an indication for active surveillance in some patients. Active surveillance means that the patient will either have delayed treatment or no treatment at all.

NOTE - Active surveillance is not generally recommended for healthy younger patients because while there is some evidence to suggest such approach if the lesion is less than 1 cm; more data is needed before adopting this as a standard protocol in this patient group. Another issue of concern, when considering active surveillance in young patients, is the number of scans they will require over a lifetime, which is associated with considerable exposure to radiation.

(Options A and B) Tumor resection is the management of choice for small renal masses that are suspected of malignancy. Such masses are solid, mixed solid, or cystic renal lesions that enhance with contrast.

If possible, nephron-sparing surgery or partial nephrectomy rather than a total nephrectomy is performed to allow for preservation of renal function. However, in the following situation total nephrectomy is the treatment of choice:
* Tumor size ≥7 cm
* Those tumors that have a more central location
* Suspected lymph node involvement
* Tumor with associated renal vein or inferior vena cava (IVC) thrombus
* Direct extension into the ipsilateral adrenal gland

Even in the presence of the above, patients with any of the following conditions must have partial rather than total nephrectomy:
* A solitary kidney
* Multiple, small, and/or bilateral tumors
* Patients with or at risk for chronic renal disease

(Options D) The role of renal biopsy is controversial in the setting of a small renal mass. In particular, there is disagreement as to whether it is necessary to biopsy these lesions before planning management and in what settings. Circumstances under which a renal biopsy might be considered are where the patient is not a surgical candidate (controversial), life expectancy is ≤5 years (controversial), or the patient requests a definite diagnosis before proceeding with the surgery. Biopsy is not recommended for patients who are candidates for active surveillance.

NOTE - With available evidence, it may be safe to assume that routine discontinuation of the antiplatelet therapy is not necessary in low risk patients planned to undergo percutaneous renal biopsies.

(Option E) Recommended interval for active surveillance with CT scan or MRI is 6-12 months, not 3 months.

How well did you know this?
1
Not at all
2
3
4
5
Perfectly
14
Q

A 77-year-old man presents to your clinic with vague abdominal pain. Among initial laboratory tests, hematuria is notable. A repeated urinalysis confirms hematuria. You order a 4-phase contrasted abdominal CT scan that shows a 1cm complex mass in the lower pole of the right kidney. He is otherwise asymptomatic, enjoys a healthy life, and do not have any comorbidities.

Which one of the following is the most appropriate inital management option for him?

A. Repeat the CT scan in 12 months.
B. Nephrectomy.
C. CT-guided percutaneous biopsy.
D. Reassurance.
E. Thermal ablation

A

Correct Answer Is B.

Abdominal pain and hematuria in the presence of a renal mass is renal cell carcinoma (RCC) until proven otherwise. Small renal masses (< 4 cm) can be found incidentally, or during evaluation for urinary symptoms, often by ultrasound as the initial imaging modality. Once a mass is found in the kidney on ultrasound, a 4-phase contrasted CT scan should follow for further evaluation if contrast media is not contraindicated.

Solid or complex masses (a lesion demonstrating both cystic and solid structures), as well as cystic masses enhanced with contrast are suspected to be malignant and treatment is warranted.

NOTE - By definition, a renal lesion <4 cm in its largest diameter that shows contrast enhancement on abdominal imaging is a small renal mass. These lesions can be solid or complex cystic. Simple cysts not enhancing with contrast are excluded.

If the patient is a good candidate for surgical resection and has a life expectancy >5 years, surgical resection of the small renal mass would be the most appropriate initial management. This patient is otherwise fit and healthy with no comorbid condition precluding him from surgical tumor resection. Partial nephrectomy is the procedure of choice for masses less than 7cm that are not centrally located. This is to preserve as much renal function as possible.

(Option A) Active surveillance with CT scan or MRI in 6- to 12-months intervals is recommended for patients with small renal masses who are not good candidates for surgical resection (e.g., have a comorbid condition), or have a life expectancy of ≤5 years. Renal impairment may also be an indication for active surveillance in some patients. Active surveillance means that the patient will either have delayed treatment or no treatment at all. This patient has no comorbid condition precluding him from tumor resection.

(Option C) The role of renal biopsy is controversial in the setting of a small renal mass. There is disagreement as to whether it is necessary to biopsy such lesions before planning management and in what settings. Circumstances under which a renal biopsy might be considered are where the patient is not a surgical candidate (controversial), life expectancy is ≤5 years (controversial), or the patient requests a definite diagnosis before proceeding with the surgery. Biopsy is not recommended for patients who are candidates for active surveillance.

(Option D) With this clinical picture, the chances are that the tumor is malignant; therefore, reassurance cannot be given unless investigations exclude malignancy.

(Option E) Ablation of renal mass can be performed with either freezing (cryoablation) or heat (radiofrequency or microwave ablation). Using a percutaneous approach, a needle is used to ablate the tumor with heat of freezing. After ablation, all patients should undergo surveillance to assess for residual or recurrent tumor that would require additional therapy. Although initial studies have shown favorable short-term results with thermal ablation, long-term outcomes are yet to be studied. Thermal ablation is reserved for patients with a small RCC who are not candidates for surgery. In addition, ablation is a treatment option for RCC after partial nephrectomy, those with a unilateral or transplanted kidney, or for patients at risk for multiple RCCs over their lifetime.

How well did you know this?
1
Not at all
2
3
4
5
Perfectly
15
Q

A 62-year-old man is referred to your clinic for a health checkup by his insurance company. An ultrasound of the kidney, ureter, and bladder reveals a 3-cm heterogeneous mass in the upper pole of his right kidney. A contrast-enhanced CT scan is ordered that confirms the presence of a contrast-enhancing cystic mass. Which one of the following would be the most appropriate management for this patient?

A. Total nephrectomy.
B. Review in 12 months.
C. Percutaneous biopsy.
D. Partial nephrectomy.
E. Thermal ablation.

A

Correct Answer Is D.
Rising use of imaging studies as a means of diagnosis has increased the rate at which small renal masses are detected. Incidentally-found small renal masses (incidentalomas) are now a common clinical scenario. Recent data have revealed that over 50% of renal cell carcinomas (RCCs) are incidentally found, and the classic presentation with a triad of gross hematuria, flank pain, and abdominal mass is not commonly encountered.

The majority of incidentally-detected renal masses are benign simple cysts that require no further work-up. However the following renal masses are more likely to be malignant:
* Completely solid renal masses
* Mixed solid and cystic renal lesions
* Cystic lesions that enhance with contrast

By definition, a renal lesion <4cm in its largest diameter that shows contrast enhancement on
abdominal imaging is a small renal mass. These lesions can be solid or complex cystic. Simple cysts not enhancing with contrast are not considered renal small masses.

For small renal masses, surgical resection is the most appropriate option if life expectancy is >5 years and the patient is a good candidate for surgery. This man has a 3-cm complex tumor and needs surgical resection of the tumor as the most appropriate management option.

For renal masses smaller than 7-cm in size, nephron-sparing surgery, or partial nephrectomy rather than a total nephrectomy is performed to allow for preservation of renal function. This patient with a 1cm mass is most likely to benefit from partial nephrectomy as the most appropriate management option.

(Option A) Total nephrectomy is the treatment of choice in the following situations:
* Tumor size ≥7 cm
* Tumors with a more central location
* Suspected lymph node involvement
* Tumor with associated renal vein or inferior vena cava (IVC) thrombus
* Direct extension into the ipsilateral adrenal gland

Even in the presence of the above, patients with any of the following conditions must have partial rather than total nephrectomy:
* A solitary kidney
* Multiple, small, and/or bilateral tumors
* Patients with or at risk for chronic renal disease

(Option B) Active surveillance with CT scan or MRI in 6- to 12-month intervals is recommended for
patients with a small renal mass, who are not good candidates for surgical resection (e.g., have a comorbid condition), or have a life expectancy of ≤5 years. Renal impairment may also be an indication for active surveillance in some patients. Active surveillance means that the patient will either have delayed treatment or no treatment at all. This patient has no comorbid condition precluding him from tumor resection.

(Option C) The role of renal biopsy is controversial in the setting of a small renal mass. In
particular, there is disagreement about necessity to biopsy these lesions before planning management. Circumstances under which a renal biopsy might be considered are:
* The patient is not a surgical candidate (controversial)
* Life expectancy is ≤5 years (controversial)
* The patient requests a definite diagnosis before proceeding with the surgery.

NOTE - Biopsy is **not recommended **for patients who are candidates for active surveillance.

(Option E) Ablation of renal mass can be performed with either freezing (cryoablation) or heat
(radiofrequency or microwave ablation). Using a percutaneous approach, a needle is used to ablate the tumor with heat or freezing. After ablation, all patients should undergo surveillance for assessment of residual or recurrent tumor that would require additional therapy. Although initial studies have shown favorable short-term results with thermal ablation, long-term outcomes are yet to be studied. Thermal ablation is reserved for patients with a small RCC, who are not candidates for surgery.

Ablation is also considered for:
* Treatment of residual RCC after partial nephrectomy
* Those with a unilateral or transplanted kidney
* Patients at risk for multiple RCCs over their lifetime.

How well did you know this?
1
Not at all
2
3
4
5
Perfectly
16
Q

A 69-year-old man presents to your GP clinic with complaint of uriary incontinence. He explains that he cannot make it to the bathroom once he feels the urge to pass urine. He denies any dysuria.

Which one of the following could be the most likeky underlying cause to this presentation?

A. Benign prostatic hyperplasia (BPH).
B. Urinary tract infection (UTI).
C. Detrusor instability.
D. Increased intraabdominal pressure.
E. Nephrolithiasis.

A

Correct Answer Is C.

The incontinence described in the scenario is characteristic of urge incontinence. In this type of incontinence, there is sudden and strong urge to pass urine. The patient often is not able to make it to the bathroom and wet themselves. An overactive bladder caused by detrusor instability is the most common etiology of urge incontinence and the most likely cause to this presentation. An irritable or unstable bladder are synonymous terms used to describe the pathology. Patients with detrusor instability have involuntary bladder contractions, resulting in a sudden urge to urinate.

(Option A) BPH usually causes overflow urinary incontinence rather than urge incontinence.

(Option B) Although UTI is a cause of frequency and urgency due to irritation of the bladder, detrusor instability of unknown etiology is the most common cause of urge incontinence.

(Option D) Increased intraabdominal pressure is a cause of urine overflow in stress incontinence not in urge incontinence.

(Option E) A stone in the bladder can irritate the bladder and cause frequency, urgency, and urge incontinence but idiopathic detrusor instability remains the most common cause of urge incontinence.

How well did you know this?
1
Not at all
2
3
4
5
Perfectly
17
Q

A 70-year-old man presents to the Emergency Department with complaint of intermittent loin pain for the past 48 hours. On examination, he is afebrile and there is no tenderness over the loin, flank or costovertebral angle. A urine test shows 3+ hematuria with negativity for protein and nitrite. Which one of the following is the most appropriate next step in management?

A. Abdominal X-ray.
B. Intravenous pyelogram (IVP).
C. Abdominal CT scan.
D. Retrograde pyelography.
E. Urine culture.

A

**C. Abdominal CT Scan **

The scenario is highly suggestive of kidney stone(s) (nephrolithiasis) as the most likely diagnosis.

Nephrolithiasis affects one in 10 persons in Australia. Although most stones pass spontaneously, some conditions require intervention. These conditions include:
* Continuous pain
* Renal impairment
* Urinary tract infections

For every patient presenting with a clinical picture suggestive of urinary stones, investigations should be considered to determine the location, number and size of the stone(s) which influence its likelihood of spontaneous passage.

Non-contrast CT scan of the abdomen for assessment of the kidneys, ureters and bladder (CT-KUB) is the investigation of choice to consider. CT-KUB has a sensitivity and specificity of approximately 100% for evaluation of urinary stones irrespective of stone type. It also provides an opportunity for an alternative diagnosis if no stone(s) is visualized. It is important to note that an additional KUB plain film (x-ray) should be obtain at the same presentation. If the stone is visible on plain KUB, this can be used for follow up imaging, with a lower radiation dose. Only calcium-containing stones are visible on plain KUB X-ray, and this information also has implications for stone management.

(Option A) As mentioned above, a KUB x-ray is performed for reducing the radiation exposure associate with CT scanning if the stone is visualized. Otherwise, this modality is not sensitive enough to establish a diagnosis.

(Option B) Intravenous pyelogram (IVP) is used rarely in the diagnosis of ureteric colic due to the high sensitivity and specificity of non-contrast CT KUB. However, a contrast study (CT urogram) may be helpful sometimes for treatment planning purposes. Retrograde pyelography (option C) serves the same purpose as does IVP but the contrast material is pushed up through a urinary catheter. This test has no role for evaluation of patients with real colic.

(Option E) This patient has no signs, symptoms, or dipstick urine exam findings suggestive of urinary tract infection. If that was the case, a urine culture and sensitivity would have been indicated for further management.

NOTE - Ultrasound is generally reserved for assessment of loin pain in pregnant women or for follow-up of patients with uric acid stones, which cannot be seen on a plain KUB X-ray due to their radiolucency. Ultrasound may also be used in follow-up to ensure that signs of obstruction such as hydronephrosis have resolved after conservative management of ureteric colic, or to assess the stone size in asymptomatic patients. There is no role for magnetic resonance imaging (MRI) in evaluation or management of urolithiasis.

18
Q

An 85-year-old woman presents with complaint of leakage of small amounts of urine when she laughs, coughs, or sneezes. On examination, she has a reducible rectocele. As the most appropriate initial step in management you advise bladder training and pelvic floor muscle exercise that fails to improve her symptoms. Which one of the following is the most appropriate next step in management?

A. Vaginal cones.
B. Imipramine.
C. Surgical repair of the rectocele.
D. Bladder neck suspension.
E. Vaginal estrogen cream.

A

Correct Answer Is D.

The scenario is a typical case of stress urinary incontinence. Stress incontinence is the most common cause of urinary incontinence in younger women and the second most common cause in older women (after mixed incontinence), and occurs when an increase in intraabdominal pressure
overcomes sphincter closure mechanisms in the absence of a bladder contraction.

Pelvic floor muscles exercise is the initial therapy in women with stress and mixed incontinence, and also useful, in combination with bladder training, for women with urge incontinence. If pelvic floor muscle training fails, the mainstay of treatment is surgery. Surgery has high cure rates for stress urinary incontinence, even in elderly women. A vaginal or abdominal approach can be used for surgical treatment. Vaginal procedures include mid-urethral sling (insertion of a tension-free vaginal tape under the urether), bladder neck sling (suspension), or injection of urethral bulking agents. Burch’s retropubic colposuspension is an abdominal procedure for stress urinary incontinence and is more invasive prcedure compared with other surgical methods.

NOTE - Insertion of a tension-free vaginal tape (midurethral sling) was the preferred correct answer if among options. This procedure is simple and minimally invasive.

(Option A) Methods such as supervision of the pelvic floor exercise to check if the appropriate pelvic muscles are contracted efficiently, biofeedback, and vaginal weighted cones are used in combination with pelvic floor muscles exercise to achieve a better therapeutic outcome. Vaginal cones are weighted devices designed to increase the strength of the pelvic floor muscles. The cones are available in sets of 5, with identical shape and volume but increasing weights (i.e., 20 g, 32.5 g, 45 g, 60 g, and 75 g). As a progressive resistive exercise program, a single cone is inserted into the vagina and held in place by tightening the levator ani muscles for as long as 15 minutes.

As the levator ani muscles become stronger, the exercise duration may be increased to 30 minutes.
The intravaginal weight provides the sensory feedback for the desired pelvic muscle contraction. A 2013 systematic review and meta-analysis of 23 small randomized trials found some evidence that cones have increased efficacy over no active treatment, but inconclusive evidence that they provide increased efficacy over standard pelvic floor muscle exercise. An option suggesting addition of vaginal cones to the therapy might be correct but with failed pelvic floor muscle exercise, this method alone is not very likely to add therapeutic benefits. Moreover, vaginal cones are not effective in management of stress incontinence associated with pelvic organ prolapse.

(Option B) The role of drug therapy in stress incontinence is very limited. Duloxetine (a serotonin and noradrenaline reuptake inhibitor) has some effects on increasing bladder outlet resistance and
is effective in controlling mild urinary stress incontinence in women, but it is not approved for this indication in Australia.

(Option C) Currently, the complaint of this patient is urinary incontinence not the rectocele, which is an incidental finding. Women with pelvic organ prolapse, who are asymptomatic, do not require surgical repair. Moreover, in many women with pelvic organ prolapse and stress urinary
incontinence, surgical repair of the pelvic organ prolapse (e.g., rectocele) results in worsening of the urinary incontinence.

(Option E) Topical estrogen therapy has been used, as a supplement to pelvic floor exercise and bladder training, in postmenopausal women with urge and mixed incontinence. It is possible that urinary urgency and frequency in urge incontinence are symptoms of urogenital atrophy in older
postmenopausal women. There is good evidence that low-dose topical vaginal estrogen therapy improves or even may reverse the symptoms and cytologic changes of urogenital atrophy.

Systemic estrogen (with or without progesterone) should not be used to treat urinary incontinence because it can worsen the condition. Evidence supporting use of estrogen in patients with stress urinary incontinence is insufficient.

19
Q

A 70-year-old woman come to your clinic with complaint of urinary incontinence. She describes her
problem as leakage of small amounts of urine when she laughs, sneezes or coughs, as well as during exercise. Which one of the following is the** most appropriate next step** in management?

A. Pelvic floor mucles exercise.
B. Tension-free vaginal tape.
C. Burch’s colposuspension.
D. Vaginal weighted cones.
E. Topical estrogen cream

A

Correct Answer Is A.

The description is typical for genuine stress urinary incontinence (SUI) as the most likely diagnosis. For women, who present with SUI or mixed incontinence, pelvic floor exercise is the most appropriate management option to consider first.

Bladder training is the most important component of treatment in mixed incontinence with pronounced symptoms related to an overactive bladder. Pelvic floor muscle exercise strengthens the muscular components of the urethral closure mechanism. The basic recommended regimen is 3 sets of 8 to 12 slow velocity contractions sustained for 6 to 8 seconds each, performed 3 or 4 times a week and continued for at least 3 months.

(Options B and C) Tension free vaginal tape and Burch’s colposuspension are more invasive procedures considered for patients who fail to respond to conservative management strategies.
Tension free vaginal tape is a minimally invasive procedure that is performed by transcutaneous
placement of a mesh around and under the mid-urethra. It does not elevate the urethra and rather
forms a resistant platform against intraabdominal pressure. In Burch’s procedure, an intraabdominal approach is used to lift the bladder neck upwards using stitches.

(Option D) Weighted vaginal cones are used as supplement to pelvic floor muscle exercise. The
patient inserts the cone in her vagina and uses pelvic muscle contractions to hold it in place during activity. Although small trials show efficacy compared with no treatment in younger women, it is unclear if outcomes are comparable to other therapies. A 2013 systematic review and metaanalysis of 23 small randomized trials found some evidence that cones have increased efficacy over no active treatment, but inconclusive evidence that they provide increased efficacy over
standard pelvic floor muscle exercise.

(Option E) Topical estrogen therapy has been used, as a supplement to pelvic floor exercise, in
postmenopausal women with urge and mixed incontinence. It is possible that urinary urgency and
frequency in urge incontinence are symptoms of urogenital atrophy in older postmenopausal women. There is good evidence that low-dose topical vaginal estrogen therapy may reverse the symptoms and cytologic changes of urogenital atrophy. However, oral estrogens (with or without progesterone) have been shown to worsen urinary incontinence, and should be avoided. Evidence supporting the use of estrogen for patients with stress urinary incontinence is insufficient.

20
Q

A 75-year-old woman presents with leakage of urine when she laughs, sneezes, coughs or lifts heavy weights. She also complains that she feels the urge to pass urine 10-12 times a day, and half of time she cannot make it to the bathroom. Urgency appears to be more troublesome for her. A urine examination is negative for infections. Ultrasound scan of the bladder, ureter and kidneys is inconclusive, except a residual volume of 125cc. Which one of the following is the most appropriate treatment option for her?

A. Anticholinergic medications.
B. Anterior colporrhaphy.
C. Retropubic suspension of the bladder.
D. Pelvic floor muscle exercise.
E. Bladder training.

A

Correct Answer Is E.

This woman has signs and symptoms of both stress incontinence and urge incontinence, namely mixed incontinence. However, she is annoyed mostly by the urge. For women with urge continence as the most bothersome symptom, bladder training would be the most appropriate management option to consider first. The goals are:
* Establishing a baseline with a bladder diary
* Establishing a voiding interval
* Gradually increasing the voiding interval in increments such as 2–5 minutes over a prolonged time period aiming to void every 3 hours

Other important measures to consider are lifestyle modifications such as decreased fluid intake, weight reduction and avoiding certain foods and beverages that have diuretics effects (e.g., alcoholic beverages, caffeine, etc).

(Option A) Anticholinergics, combined with continued bladder training, are an option when urge incontinence does not respond to physical or behavioral therapies. A trial of 4 to 6 weeks is used. For those who respond, risk and benefits are balanced at 6 months to determine whether treatment should be continued. Patients should be advised about the adverse effects of anticholinergics such as dry mouth and constipation, and how to manage them.

(Option B) Anterior coloprrhaphy is a repair of the front vaginal wall to treat cystocele and can help women who suffer from urinary incontinence. Without a cystocele, bladder neck suspension is the
treatment of choice.

(Option C) Retropubic suspension of the bladder is a more invasive procedure used for treatment of stress urinary incontinence in patients who has failed to respond to more conservative measures such as pelvic floor exercise.

(Option D) Pelvic floor muscle exercise (e.g. Kegel exercise) is the most important initial conservative management to consider for patients with genuine stress incontinence, and mixed (both stress and urge) urinary incontinence when stress incontinence is the more prominent complaint. Badder training can be added for the urge incontinence.

21
Q

A 65-year-old woman presents to your practice with complaint of urinary incontinence that has restricted her social activities. She feels the urge to urinate while outside and often cannot make it to a bathroom. She also mentions that every time she puts the key into the door, it is like that her bladder wakes up and get irritated. Which one of the following would be the most appropriate advice for her at this stage?

A. Pelvic floor muscles exercises.
B. Anticholinergic medications.
C. Bladder training.
D. Vaginal cones.
E. Bladder neck suspension.

A

Correct Answer Is C.

The case scenario represents a typical case of urge urinary incontinence. For women with mild to
moderate stress, urge and mixed urinary incontinence, behavioral and physical therapy should be the initial treatment. It also can be an option for patients with more severe symptoms. These
therapies include bladder training and pelvic floor muscle exercise (Kegel exercise). Although these two are often used in combination, pelvic floor muscle exercises are recommended initially for genuine stress incontinence and the stress component of mixed incontinence. On the other hand, bladder training is the first line-treatment for urge incontinence. It should be noted, however, that
these two are usually advised in combination. This woman has urge incontinence; therefore, bladder training is first-line option for her treatment.

The goals of bladder training are:
* Establishing a baseline with a bladder diary
* Establishing a voiding interval
* Gradually increasing the voiding interval in increments such as 2–5 minutes over a prolonged time period aiming to void every 3 hours

(Option A) Pelvic floor muscles exercise is also an important part to consider in conservative
management in combination with bladder training; however, not as effective as bladder training in urge incontinence.

(Option C) Anticholinergics, combined with continued bladder training, are an option when urge incontinence does not respond to physical or behavioral therapies. A trial of 4 to 6 weeks is used. For those who respond, risk and benefits are balanced at 6 months to determine if treatment should be continued. Patients should be advised about the adverse effects of anticholinergics such as dry mouth and constipation, and how to manage them.

(Option D) Vaginal cones are used as supplement to pelvic muscle exercise for treatment of stress urinary incontinence. The patient inserts the cone in her vagina and uses pelvic muscle contractions to hold it in place during activity. This method is not effective for treatment of urge
incontinence. It does not seem effective in treatment of stress incontinence either if used alone.

(Option E) Bladder neck suspension is a more invasive method for treatment of stress incontinence
in patients who do not respond to more conservative measures. It is not used for treatment of patients with urge incontinence.

22
Q

A 58-year-old woman presents to your practice with the complaint that she loses control of her ‘waterworks’ when she puts her key in the front door. She denies urinary leakage associated with laughing, coughing, or sneezing. Which form of urinary incontinence is she most likely to have?

A. Incontinence from a urinary fistula.
B. Stress incontinence.
C. Urge incontinence.
D. Incontinence secondary to a neuropathic bladder.
E. Overflow incontinence.

A

Correct Answer Is C.

The patient’s description of incontinence is consistent with urge incontinence as the most likely diagnosis. Urge incontinence occurs when the patient is unable to delay micturition. The condition may be precipitated by various triggers such as the sound of running water or placing a key in the door when arriving home.

(Option A) urinary fistula is associated with continuous dribbling or leakage of small amounts of
urine on effort.

(Option B) Stress incontinence is associated with leakage of usually small amounts of urine during
activities that cause the intra-abdominal pressure to increase e.g., coughing or sneezing, which this
patient denies.

(Option D) Neuropathic (neurogenic) bladder refers to bladder dysfunction secondary to neurologic
damage. Symptoms can include overflow incontinence, frequency, urgency, urge incontinence, and urinary retention. Although a neurogenic bladder may lead to urge incontinence, it is the cause
rather than the type of the incontinence.

(Option E) Overflow incontinence is due to obstruction and may be secondary to uterovaginal
prolapse or a hypotonic bladder as in a neuropathic bladder.

23
Q

A young couple present to your rural GP clinic because the wife has been unable to conceive despite regular timely unprotected sex for the past 12 months. Assessments on the wife’s side reveals no cause. On male’s side, a semen analysis shows azoospermia. Which one of the following is the most important next test for this man?

A. Serum FSH level.
B. Serum LH Level.
C. Serum testosterone level.
D. Anti-sperm antibodies.
E. Testicular ultrasound.

A

Correct Answer Is A. Serum FSH level.

A 2.5 times or more increase in serum FSH level indicates irreversible testicular failure. FSH is the most important endocrine test in assessment of male infertility.

Serum testosterone level is a helpful test in making diagnosis of hypogonadism; however, its level varies in different age groups and low level does not necessarily indicate irreversible testicular failure. In infertile men, testosterone levels are often within normal range.

Other investigations including LH, inhibin, antisperm antibodies, chromosome analysis, and testicular ultrasound all are helpful in assessment of male infertility, but serum FSH remains is the most important hormonal test, especially in patients with azoospermia.

24
Q

A 58-year-old man is brought to the emergency department with an ambulance due to confusion at home notice by his wife. He is a known case of chronic kidney disease (CKD), hypertension and congestive heart failure (CHF). He is currently on carvedilol 6.25mg BD, Ramipril 10mg BD, aspirin 100mg daily and atorvastatin 10mg daily. On examination, he has a blood pressure of 148/97mmHg, pulse rate of 98bpm, respiratory rate of 21 breaths per minute and temperature of 37.3°C. He is confused and disoriented to time, place and person. Laboratory tests for him are
ordered, of which the results for his urea and electrolytes are as follows:

Urea: 18mmol/L (2.9-8.2 mmol/L)
Creatinine: 9 mmol/L (up to 0.11mmol/L in males; up to 0.9mmol/L in males)
Calcium: 3.1 mmol/L (2.10-2.50mmol/L)
Phosphate: 3.5 mmol/L (1-1.5mmol/L)
Sodium: 128mmol/L (135-145mmol/L)
Potassium: 5.7 mmol/L (3.5-5.1mmol/L)

Which one of the following could be the most likely cause of his confusion?

A. Hyperphosphatemia.
B. Hyponatremia.
C. Hyperkalemia.
D. Uremia.
E. Hypercalcemia.

A

Correct Answer Is D.

Of the options, elevated levels of serum urea are most likely to have contributed to this presentation.

Given the very high levels of serum urea and creatinine in this patient, he is most likely suffering
from uremic encephalopathy. Early symptoms of uremic encephalopathy include the following:
* Anorexia
* Nausea
* Restlessness
* Drowsiness
* Diminished ability to concentrate
* Slowed cognitive functions
* More severe signs and symptoms of uremic encephalopathy include the following:
* Vomiting
* Emotional volatility
* Decreased cognitive function
* Disorientation
* Confusion
* Bizarre behavior
* Stupor, coma

In a patient with CKD, a number of electrolyte disturbances occurs. Some of the most important of
such disturbances are:

Hyperkalemia (option C) – in patients with CKD, hyperkalemia is one of the most common and lifethreatening electrolyte aberrancies. The inability of failed kidneys in renal excretion of potassium is the main reason for hyperkalemia. Hyperkalemia is associated with non-specific findings of fatigue,
muscular problems and cardiac rhythm disturbances. Confusion is not a prominent feature.
Furthermore, a chronically increased serum potassium level of 5.7 mmol in a patient with CKD is
not of a big concern and is usually well-tolerated without any serious direct symptoms.
Hyponatremia (option B) – hyponatremia is another finding in patients with patients with CKD
mostly due to inability of kidneys to retain sodium. This patient also has CHF and probably additional volume overload leading to dilution hyponatremia. Although both hyponatremia and hypernatremia have CNS presentations such lethargy, confusion, or in severe cases, convulsions or even coma, a serum sodium level of 128 mmol/L, which is very likely to have occurred chronically, is very unlikely to result in confusion. Hyponatremia in this patient does not justify the clinical presentation of confusion in this man.

Hypercalcemia and hyperphosphatemia (options A and E) – deranged bone metabolism in patients
with CKD results in these two electrolyte abnormalities. Hypercalcemia is associated with a long list clinical findings such as:
CNS:
* Lethargy
* Weakness
* Confusion
* Coma

Renal system:
* Polyuria
* Nocturia
* Dehydration
* Renal stones
* Renal failure

Gastrointestinal:
* Constipation
* Nausea
* Anorexia
* Pancreatitis
* Gastric ulcer

This man has hypercalcemia, which can justify his confusion. However, it should be noted that
hypercalcemia in patients with CKD is chronic and often well-tolerated. As a general rule, chronic
electrolyte derangements are less likely to cause acute symptoms as the body adopts strategies to
cope with the condition in time.

25
Q

A 65-year-old man presents to your GP clinic with long-standing history of difficulty starting voiding
and terminal dribbling and, recently, nocturia. Abdominal examination reveals a dull mass over the
suprapubic area. On digital rectal exam (DRE), the prostate is enlarged but smooth with palpable
median sulcus. Which one of the following is the most appropriate next step in management of this patient?

A. Serum creatinine level.
B. Transrectal ultrasonography (TURS).
C. CT scan of the abdomen.
D. PSA.
E. Urine cytology.

A

Correct Answer Is A.

This patient has characteristic features of** lower urinary tract symptoms (LUTS)**. LUTS can present with voiding symptoms (bladder emptying) such as weak urine stream, hesitancy (difficulty starting urination) or intermittency of follow, or with storage (bladder filling) symptoms such as urgency,
daytime frequency and nocturia.

Predominance of storage symptoms requires exclusion of conditions such as primary bladder pathology/ malignancy, diabetes mellitus, and medications with diuretic effects. Where nocturia is the presenting symptom, nocturnal polyuria should be excluded too.

Based on the international guidelines, it is important to determine the severity of LUTS and to identify the complicating factors such as urinary retention, macroscopic hematuria, urinary tract infections or a personal or family history of prostate cancer.

For every patient presenting with LUTS, initial investigations should be considered to exclude
sinister causes LUTS or complications associated with bladder outflow obstruction. These investigations include:

  • Urinalysis - excluding hematuria, proteinuria, or pyuria - follow up with urine culture is required if there is abnormality on urinalysis
  • Serum creatinine and estimated glomerular filtration rate (eGFR) - this test is required for
    exclusion of renal injury from primary renal dysfunction or high-pressure bladder outflow
    obstruction.
  • Urinary tract ultrasound - assessment of prostate volume, bladder wall and residual urine and
    exclusion of hydronephrosis
  • Prostate specific antigen to exclude prostate cancer - most guidelines recommend the use of
    serum PSA only if prostate cancer diagnosis will influence management or if the test will assist in decision making

Urinary symptoms of this man and the presence of an enlarged prostate on DRE make benign prostatic hyperplasia (BPH) the most likely diagnosis. Additionally, the dull mas over the suprapubic area is very likely to be a distended bladder due to urinary outflow obstruction caused by the enlarged prostate. In this situation and of the options, a serum creatinine level [and calculation of the estimated glomerular filtration rate (eGFR)] would be the most important next step to exclude renal injury caused by the obstruction.

If an option, transabdominal ultrasound scan of the bladder was another important test to consider first to assess the residual volume, prostate size and other possible urinary tract anomalies.

(Option B) While transabdominal ultrasound is a very important investigation for this patient, transrectal ultrasound is not necessary at this stage. However, it might be applied in patients who are suspected of having prostate cancer for further assessment and as a guide for biopsy.

(Option C) CT scan is not routinely recommended for patients with LUTS, unless complicating features are suspected. An example could be for staging in a patient with bladder or prostate cancer as the etiologic cause of LUTS.

(Option D) As mentioned earlier, PSA is among the investigations for male patients with LUTS to
exclude for prostate cancer; however, assessment of renal function takes precedence over PSA in this patient.

(Option E) Urine cytology is a test considered for patients with suspected urothelial carcinoma such as those with hematuria who have risk factors like history of smoking or exposure to certain chemicals. It is not a routine test for patients with LUTS unless there are alarming features.

26
Q

Tom, 67 years old, presents to your GP clinic with complaint of urinary retention since this morning. He mentions that he has had difficulty starting voiding and terminal dribbling. On examination, he is in mild distress. All his vital signs including the temperature are within normal limits. On abdominal examination, a slightly tender dull mass is felt up to 2-finger breadth above his symphysis pubis. A digital rectal exam (DRE) reveals an enlarged prostate with normal consistency, no nodularity and a palpable median sulcus. You insert a urinary catheter for immediate relief of symptoms. Which one of the following is the most appropriate next step in management?

A. Order serum PSA level.
B. Perform a bladder scan.
C. Arrange for abdominal CT scan.
D. Arrange for serum creatinine measurement.
E. Obtain a sample for urine cytology.

A

Correct Answer Is D. Arrange for serum creatinine measurement.

The history of difficulty starting voiding (urinary hesitancy), terminal dribbling, and recently, acute urinary retention, in combination with an enlarged but otherwise normal prostate on DRE, makes benign prostatic hyperplasia (BPH) and, subsequently, urinary retention the most likely diagnosis in Tom.

BPH is chronic in nature and gradually results in urinary outflow obstruction and its complications such as hydronephrosis and kidney damage. Therefore, it is very important to consider initial investigations for every patient who presents with lower urinary tract symptoms (LUTS) to exclude sinister causes of such symptoms and complications of obstructions.

At this stage, it is crucial that possible impaired kidney function due to obstruction is considered and excluded promptly with a serum creatinine level and estimated glomerular filtration rate (eGFR) as the most appropriate option among others.

(Option A) Prostate specific antigen (PSA) is elevated not only in prostatic cancer, but also in benign conditions such as BPH and prostatitis. It is controversial whether PSA should be routinely assessed in patients presenting with LUTS due to a prostatic pathology. According to the Royal Australian College of General Practitioners (RACGP), PSA may be checked, after appropriate counselling, to assess the risk of prostate cancer. In general, most guidelines recommend the use of PSA if prostate cancer diagnosis will influence the management or if the test will help with decision making. DRE findings in Tom favor BPH rather than prostate cancer in which a hard, irregular prostate with obliterated median sulcus is expected. A PSA may or may not be considered for him after appropriate counselling, but it is not necessary at this stage.

(Option B) A bladder scan uses a noninvasive, portable ultrasound device that provides a virtual 3D image of the bladder and the residual volume. The main reason to perform a bladder scan is to measure the residual volume. In this patient for who a urinary catheter has emptied the bladder already is of no use at this stage. A formal ultrasound of the urinary system (different from bladder scan), however, is very important in patients with LUTS, especially with obstructive symptoms, to exclude complications of obstruction such as hydronephrosis or other abnormalities.

(Option C) CT scan is not routinely recommended for patients with LUTS unless complicating features are suspected. An example could be for staging in a patient with bladder or prostate cancer as the etiologic cause of LUTS.

(Option E) Urine cytology is appropriate if there is a suspicion of urothelial carcinoma. Examples include patients with gross hematuria or unexplained persistent (present on 2 of 3 analysis) microscopic hematuria and risk factors for urothelial carcinoma such as smoking, age>40 years or exposure to chemicals.

NOTE – although not an option, urinalysis (UA) and, if there is evidence of urinary tract infection, urine culture should be performed in all patients with LUTS.

27
Q

An 82-year-old woman presents to your clinic with signs and symptoms suggestive of stress urinary incontinence. On examination, there is no pelvic organ prolapse. Which one of the following is the most appropriate next step in management?

A. Urodynamic studies.
B. Ascending urethrogram.
C. Surgery.
D. Pelvic floor muscle exercise.
E. Bladder training.

A

Correct Answer Is D. Pelvic floor muscle exercise.

A thorough history, physical examination, and urinalysis are usually sufficient to make a diagnosis of urinary incontinence and its type and etiology. The initial visit can be used to classify the type of incontinence, evaluate for potentially reversible conditions, and check a urinalysis. The initial conservative treatment should be advised and started then.

This woman has urinary incontinence. For her, advising a voiding diary, lifestyle changes such as weight loss (if obese or overweight), decreased fluid intake, avoiding diuretic foods and beverages should be considered; however, the most important component of initial management of patients with mild to moderate stress urinary incontinence will be pelvic floor muscle exercise, also known as Kegel exercise. It also can be an option for patients with more severe symptoms.

(Option A) Routine urodynamic studies are not recommended for evaluation of urinary incontinence. Urodynamic tests are the physiological diagnostic ‘gold standard’, but are invasive, expensive, require special equipment and training, are not usually necessary to make the diagnosis, and have not been found to affect outcome. These studies, however, are important when invasive therapy is planned.

(Option B) Imaging studies other than ultrasound in selected cases are not often required for diagnosis of urinary incontinence.

(Option C) Surgical procedures are used for patients with stress urinary incontinence, who have failed to respond to more conservative treatment such as pelvic floor muscle exercises.

(Option E) Bladder training should be offered as first-line treatment for overactive bladder symptoms such as frequency or urgency in patients with urge and mixed incontinence. It also may be beneficial for patients with stress incontinence in combination with pelvic floor muscle exercise.

28
Q

A 68-year-old woman presents with complaint of urinary incontinence occurring when she laughs, sneezes, coughs, or when she lifts heavy weights or does exercise. She denies urgency or frequency. On examination, a rectocele is noted. A mid-stream urine exam is ordered which is unremarkable. Which one of the following is the most appropriate next step in management?

A. Surgical repair of the rectocele.
B. Pessaries.
C. Pelvic floor exercises.
D. Review in 4 weeks.
E. Bladder training program.

A

Correct Answer Is C. Pelvic floor exercises.

For women with mild to moderate stress, urge and mixed urinary incontinence, behavioral and physical therapy should be the initial treatment. It also can be an option for patients with more severe symptoms. These therapies include bladder training and pelvic floor muscle (Kegel) exercise. Although these two are often used in combination, pelvic floor muscles exercises are recommended initially for stress or mixed incontinence as the most important arm of treatment.

On the other hand, bladder training is the first line-treatment for urge incontinence or mixed incontinence when symptoms of urge incontinence predominate. It should be noted, however, that these two are usually advised in combination.

(Option A) Pelvic organ prolapse (e.g., rectocele) can have been caused by pelvic floor weakness which is often the same etiology of stress urinary incontinence. Surgical repair of pelvic organ prolapse is only indicated for patients who are symptomatic. Urinary incontinence is not a symptom of this condition. It is caused by the same etiology of pelvic organ prolapse.

(Option B) Continence pessaries are used as adjunctive therapy for women with pelvic organ prolapse or stress urinary incontinence. This method is inexpensive, effective and safe with no or rare serious complication or contraindication. This can be used adjunctive to pelvic floor muscle exercise, but not as the most appropriate method per se.

(Option D) Reviewing the patient in 4 weeks without considering any active management is not an appropriate option.

(Option E) Bladder training should be offered as first-line treatment for overactive bladder symptoms such as frequency or urgency in patients with urge and mixed incontinence. It also may be beneficial for patients with stress incontinence in combination with pelvic floor muscles exercise, but not as the most crucial portion of treatment of urinary stress incontinence.

TOPIC REVIEW
Urinary incontinence can be categorized as:

(PICTURE: TABLE ON PAGE 1920)

29
Q

Simon, 67 years of age, has been diagnosed with troublesome benign prostatic hyperplasia (BPH), for which you referred him to a urology service. The urologist has considered transurethral resection of the prostate (TURP) as the most appropriate treatment option for him. Simon wants to know what complications this procedure may cause. In consulting him regarding TURP complications, which one of the following will you mention as the most common expected longterm complication of TURP?

A. Urethral stricture.
B. Erectile dysfunction.
C. Urinary incontinence.
D. Retrograde ejaculation.
E. Infection.

A

Correct Answer Is D. Retrograde ejaculation.

TURP is associated with different complications, of which retrograde ejaculation (backflow of the semen into the bladder during ejaculation) has been shown to be the most common long-term problem observed in 80-90% of patients.

Other complications of TURP (both short-term and long-term) include:
* Erectile dysfunction - up to 10% of men who have undergone a TURP will have problems with either having or maintaining an erection after the surgery. This can be temporary or permanent.
* Recurrent prostate enlargement - in 10% of men undergoing TURP will recur within 10 years.
* Urinary tract infection (UTI) - UTI can complicate as many as 5% of patients following TUPR.
* Bleeding - In approximately 2% of cases, persistent bleeding may occur during or after the operation. A blood transfusion may be required.
* Urinary retention - approximately 2% of patients will suffer from urinary retention after TURP.

In some cases, the bladder regains normal functions within a few weeks.

Urethral stricture - post-procedure urethral stricture is seen in approximately 4% of patients. Symptoms of a urethra stricture include straining to pass urine, spraying of urine or a “splitstream” of urine, post-void dribbling, and mild dysuria.

Urinary incontinence - some degree of urinary incontinence, mostly in urge incontinence. The condition often improves within weeks after the surgery but occasionally can become a longterm problem. It usually takes the form of urge incontinence – where you have a sudden urge to pass urine and lose control of your bladder if you do not find a toilet quickly enough.

TOPIC REVIEW

TURP syndrome
TURP Syndrome is a rare but potentially life-threatening complication of TURP. It occurs due to the absorption of the fluids used to irrigate the bladder during the operation into the prostatic venous sinuses. Signs and symptoms are varied and unpredictable, and result from fluid overload and disturbed electrolyte balance, hyponatremia in particular.

The clinical picture of TURP syndrome will vary according to its severity, and may further be influenced by the type of irrigant used. There is no classical presentation for TURP syndrome, indeed signs and symptoms are often vague and non-specific. Most of the manifestations are secondary to fluid overload.

Central nervous system manifestations:
* Restlessness
* Headache
* Nausea and vomiting
* Confusion
* Visual disturbances
* Cerebral edema
* Convulsions
* Coma

Cardio-pulmonary system manifestations:
* Bradycardia
* Hypo- or hypertension
* Tachypnea
* Hypoxia
* Cyanosis
* Pulmonary edema
* Systemic manifestations:
* Hypothermia
* Abdominal pain and distension

30
Q

A 22-year-old man presents to the Emergency Department with cough and dyspnea of one week duration, as well as fatigue, decreased urine output and dark-colored urine. Physical examination is otherwise inconclusive. Which one of the following is the single most accurate investigation to establish a diagnosis?

A. Chest X-ray.
B. Renal biopsy.
C. Anti-glomerular basement membrane antibody.
D. Urine analysis.
E. High-resolution CT (HRCT) scan of the chest

A

Correct Answer Is B. Renal biopsy.

In this patient, simultaneity of lung involvement symptoms (cough and dyspnea) and renal involvement symptoms makes Goodpasture syndrome, also called anti-glomerular basement membrane disease (anti-GBM disease) the most likely diagnosis. The disease may affect all age groups but is much more common in those younger than 30 years of age. Pulmonary manifestations of the disease include shortness of breath, cough and sometimes excessive hemopysis due to alveolar hemorrhage.

NOTE - Lung involvement, specifically alveolar hemorrhage, can be seen with other acute nephritides due to pulmonary edema, or due to lung involvement in ANCA positive and other forms of systemic vasculitis, and lupus. In the latter however, other clinical findings in addition to lung and kidney symptoms would be expected. For example, it is unlikely that lupus affects only the lungs and the kidneys, whereas Goodpasture syndrome almost always affects the kidneys and the lungs only.

For this patient a urine analysis (option D) and chest x-ray (option A) should be order as the initial workup. Urine analysis in patients with Goodpasture disease usually shows the typical picture of glomerulonephritis including proteinuria (usually not in nephrotic range), dysmorphic red cells,
white cells, and red cell and granular casts. These findings, however, indicates glomerulonephritis without delineating a specific etiology. Chest X-ray in patients with the disease shows pulmonary infiltrates which indicates pulmonary involvement but again without a specific cause. HRCT (option E) can be considered for a detailed assessment of the lungs. The criterion standard for Goodpasture disease is detection of anti-GBM antibodies either in the serum or in the kidneys.

Renal biopsy should be performed in all patients with suspected Goodpasture syndrome unless it is contraindicated because serologic assay is variable in such patients. Light microscopy of the specimen from biopsy often shows crescentic glomerulonephritis, whereas immunofluorescence microscopy reveals the pathognomonic finding of linear deposition of IgG along the glomerular capillaries and occasionally the distal tubules. In the absence of renal biopsy as an option, anti-GBM antibody (option C) assay is the second most accurate test to establish the diagnosis.

If the question asked about the most important next step or what to consider first, the correct answer urine analysis. This patient has presented with a clinical picture of acute kidney disease (AKD) and lung symptoms. In the very first step it is important to define whether the AKD is prerenal, renal, or post-renal.
- Pre-renal AKD is caused by renal hypoperfusion, such as in hypovolemia, sepsis and reduced effective arterial blood volume (e.g., heart failure, decompensated liver failure).
- Intrinsic causes of AKI include acute glomerulonephritis, acute interstitial nephritis, acute tubular necrosis (due to prolonged renal hypoperfusion), vasculitis, and nephrotoxins.
- Post-renal AKI due to obstruction of the urinary tract may be caused by kidney stones, prostatic hypertrophy or malignancy.

To distinguish the diagnosis the following investigations are required:
Urine should be collected for urinalysis, microscopy/culture/ sensitivity, red cell morphology, casts and quantification of creatinine, albumin and protein. Urinary sodium is helpful to understand the cause, low (less than 20 mmol/L) urinary sodium suggests pre-renal azotemia rather than acute tubular necrosis.

Renal ultrasonography to look for urinary obstruction.

Blood tests to look for other underlying pathology if there are suggestive features in the history, examination, and the initial urine investigations (e.g., dysmorphic red blood cells, active urinary sediment, including red and white blood cell casts). These blood tests may include C3, C4, antinuclear antibodies, extractable nuclear antigen, anti-double stranded DNA, anti-neutrophil cytoplasmic antibodies (ANCA), serum electrophoresis and serum free light chains.

This question, however, asks about the single most accurate test to reach a diagnosis, not the workup and the diagnostic algorithm.

31
Q

A 32-year-old man presents to the Emergency Department with severe right-sided colicky abdominal pain and blood in the urine. Based on the provisional diagnosis of renal stones, the patient undergoes imaging assessment with ultrasonography and abdominal CT scan revealing a 4mm stone in the vesicoureteric junction. After initial analgesia which one of the following would be the most appropriate management?

A. Extracorporeal shockwave lithotripsy (ESWL).
B. Ureteroscopy and basket stone extraction.
C. Ureteroscopy and basket stone extraction and stent placement.
D. Conservative management with analgesics and hydration.
E. Surgical removal of the stone.

A

Correct Answer Is D.

Most renal stones, even if symptomatic, can be initially managed conservatively with hydration and analgesia. The majority of stones will pass within 6 weeks. However, imaging should be performed at the first visit to allow confident stratification between cases in whom stones can be conservatively managed safely and those that should be referred for further specialist management.

Sixty percent (60%) of stones sized 5-7 mm pass with conservative management, whereas only a minority of stones >7 mm will pass spontaneously. The likelihood of spontaneous passage decreases with increasing size and with more proximal stones. The likelihood of a stone >10 mm passing spontaneously is very low. Patients should be made aware of the likelihood of passage because some patients will choose immediate surgical treatment to avoid the possibility of further episodes of renal colic, even though the stone may be of a size that should pass spontaneously.

All patients undergoing conservative management should be advised to present to the Emergency
Department if they develop symptoms of urinary tract infection (UTI), or if the pain is unbearable despite analgesia.

Renal colic can be intensely painful, as ureteric peristalsis increases back pressure behind an obstructing stone and dilates the renal capsule. Effective analgesia is important. Non-steroidal anti-inflammatory drugs (NSAIDs) provide the most effective available pain relief in renal colic. NSAIDs reduce the prostaglandin-mediated pain response, and decrease the transient increase in glomerular blood flow that accompanies acute urinary obstruction. NSAIDs can be given orally, rectally, intramuscularly, or intravenously. Intravenous route provides the most rapid onset of analgesia in renal colic. For outpatient management, oral and rectal routes provide about the same analgesia; however, given the nausea that frequently accompanies renal colic pain, many patients prefer to use NSAID suppositories.

Despite recent controversy, since the alpha-1a selective antagonist tamsulosin generally has minimal side effects (unlike the non-selective prazosin), routine use of 400 µg daily in conservatively managed of renal colic is recommended. Tamsulosin is not available on the Pharmaceutical Benefits Scheme (PBS) for treatment of urolithiasis. Alpha-1a antagonists are contraindicated in patients with end-stage hepatic or renal failure, and patients with severe
orthostatic hypotension.

Patients who are trialling conservative management should be advised to strain urine, to avoid unnecessary imaging if the stone has passed. Patients should be given a specimen jar and asked to bring the stone in to be verified; the stone should then be sent for analysis. The stone should be palpated because sometimes small blood clots may appear to be calculi on inspection.

Follow-up scans (X-ray KUB if the stone is definitely seen on initial X-ray, or CT KUB if it is not seen)
should be organized in 4 weeks after the initial visit, unless a stone has been collected and verified
by the GP or urologist.

The following patients should be referred to urologist or Emergency Department, depending on the
situation, for further assessment and treatment:
* Patients who have Stones >7 mm
* Patients who have not passes the stone after 3 weeks of conservative management – renal damage risk increases after 6 weeks of failed passage
* Patients who have ongoing pain despite adequate oral analgesia – those with unbearable pain despite oral analgesia should be directly sent to the Emergency Department
* Patients who have passed stones but have ongoing blood in the urine – for investigation of hematuria, preferably with three urine cytology tests and a CT IVP
Patients with signs and symptoms of urinary tract infection (UTI)
* Single-kidney patients.
* Patients with pre-existing renal problems
* Evidence of complete urinary obstruction such as anuria

This patient have a stone less than 7mm and has none of the above indications for referal or urgent
invasive mamagement; therefore, he can be treated conservatively for now.

Surgical removal of stones (option E) is decided by the urologist for patients who fail conservative
management, or require urgent management. Available options are ESWL (option A), ureteroscopy and use of laser to for fragmenting the stones with or without basket retrieval. A stent can be left in place for a few days (options B and C).

TOPIC REVIEW
Options for surgical removal of urinary tract stone
In Australia, there are three main surgical options for the management of urolithiasis. Except percutaneous nephrolithotomy (PCNL), other procedures are performed as day cases:

LASER LITHOTRIPSY

Laser lithotripsy can be used to treat calculi anywhere in the urinary tract, and is the most commonly performed procedure for the management of urolithiasis. A rigid or flexible scope is passed through urethra into the ureter and then a flexible laser fiber is used for fragmentation of the stone. Stone fragments may be removed with a basket or allowed to pass spontaneously. A ureteric stent is usually left in situ for a short time (usually 1–14 days) following the procedure.
Advantages:
* Good access to the whole urinary tract
* The ability to manage most stones less than 20mm in size
* Allowing s objective clearance of stone and retrieval for biochemical analysis
* Widespread availability

Risks:
* Stent symptoms
* Ureteric injury (rare)

Disadvantages:
* Sometimes requires multiple procedures
* The stent can be very bothersome for some patients

Extracorporeal shock wave lithotripsy (ESWL)

ESWL is an option, usually for stones in the renal calyces or renal pelvis that are 5–10 mm in size. A lithotripter is used with ultrasonography or X-ray guidance, under general anaesthetic, to deliver a
series of shocks to one or more stones to fracture them, allowing them to spontaneously pass with urine. ESWL is rarely used in the acute setting in Australia.

Advantages:
* It is minimally invasive
* Usually well tolerated
* A stent is not usually required

Risks:
* Bruising
* Pain
* Urinary obstruction from fragments of stone
* Hematuria
* Rarely, perinephric hematoma or significant bleeding requiring further management.

Disadvantages:
* Effectiveness decreases with increasing body mass index
* It requires expensive machinery and trained technicians to perform the procedure
* There is poor clearance of the lower pole and larger stones
* It is not appropriate for pregnant women
* It is not widely available

Percutaneous nephrolithotomy (PCNL)

PCNL is a surgical procedure in which the renal calyces are accessed via a 2 cm incision over the kidney, and a nephroscope is passed directly into the kidney. The stone is fragmented with a combination of mechanical force and direct application of ultrasound or laser. This procedure usually requires two days in hospital.

Advantages:
* Providing excellent access to large stones
* A high rate of stone clearance

Risks:
* Bleeding, renal damage and pneumothorax.

Disadvantages:
* It usually cannot access ureteric stones
* It has a higher morbidity rate
* Not every urologist performs this procedure

32
Q

A 75-year-old man presents to your practice with worsening low back pain for the last month. He denies any trauma to the back, weight loss, lower extremity pins and needles, or bladder and bowel control problems. He was concerned about prostate cancer last year and requested a prostate cancer screening with PSA the result of which was normal. There is slight tenderness over the lower back, but neurological examination is normal. Plain radiographs of the lower back show sclerotic changes in lower lumbar vertebrae and the pelvis. Which one of the following is the most appropriate next step in management?

A. Serum PSA.
B. Serum protein electrophoresis.
C. Transrectal ultrasound (TRUS).
D. Bone scan.
E. MRI of the spine.

A

Correct Answer Is A.

The sclerotic changes of the vertebrae and pelvis in an elderly male patient suggest prostatic cancer with bone metastasis as high probability. Metastatic bone lesions in prostate cancer are typically sclerotic.

For this patient, a digital rectal exam (DRE) (not an option) is the most important step to take as is repeating the serum PSA.

PSA (prostate specific antigen) is a glycoprotein produced predominantly by the prostate epithelium. It circulates in both free and bound forms. Despite modifications to the interpretation of the PSA test to improve its accuracy such as use of age-specific reference range, or attempts to make it more sensitive and specific for detection of prostate cancer, this test may still underdiagnose or overdiagnose prostate cancer. It is estimated that 20-25% of patients with prostate cancer may have a PSA level of less than 4 ng/mL which was accepted as ‘normal value’ in the past. Reports show that some patients with prostate cancer have even PSA levels of less than 1 ng/mL. On the other hand, some with PSA values greater than age-specific range reference are found to not have prostate cancer; however, such increased levels still warrant a full assessment to exclude prostate cancer.

For patients with abnormalities detected on DRE or with serum PSA greater than that expected for their age, transrectal ultrasound (TRUS) examination (option C) and prostate biopsy are performed. TRUS is not diagnostic for prostate cancer and its main use is guiding the biopsy.

Radioisotope bone scan is the most sensitive method for detecting bony metastases. Such metastases are usually distributed in the axial rather than the appendicular skeleton. However, the probability of a positive scan is very low if the PSA is less than 10 ng/mL. Based on current recommendations, bone scans in these patients are unnecessary; therefore, the decision as to bone scan (option D) is determined by the PSA levels.

CT scan and MRI (option E) are of limited use in prostate cancer because they are neither sensitive nor specific in detection of pelvic lymph node involvement and extra-prostatic disease for staging.
If the PSA level is high (>10 ng/mL) or if there is high grade disease on TRUS biopsy (Gleason score >7), lymph node sampling is usually performed at the time the patient undergoes surgery (radical retropubic prostatectomy). On the other hand, this patient does not have neurological symptoms to mandate such modalities for the assessment of back pain per se.

Serum protein electrophoresis (option B) is among investigations to consider in suspected multiple myeloma where bone lesions are lytic rather than sclerotic.

TOPIC REVIEW
The following are associated with increases serum PSA:
* Prostate biopsy
* Prostatitis
* Ejaculation within one hour before the test
* Urinary retention
* Transurethral resection of prostate (TURP)

NOTE – DRE does not significantly increase PSA, neither do alpha blockers such as prazosin or tamsulosin as it was previously thought. The following are associated with decreased serum PSA: * Finasteride * Anti-androgen therapy * Radiotherapy.

33
Q

A 26-year-old man is referred for a routine medical check by his insurance company. He has no significant medical history, and physical examination is unremarkable. As part of the assessment, a urine dipstick test is performed which is only positive for 1+ proteinuria. Other parameters are all normal. Which one of the following is the most appropriate next step in management?

A. Serum electrolytes, urea, and creatinine.
B. 24-hour urinary protein assay.
C. Perform another urine dipstick in early morning.
D. Urinary albumin to creatinine ration (ACR).
E. Urine microscopy.

A

Correct Answer Is C.

People with normal kidney function excrete less than 150 mg of protein per day in their urine, of which approximately 20 mg is albumin. Persistent protein excretion significantly above this level is a marker for kidney disease.

Proteinuria on dipstick in primary care is frequently an incidental finding and is often benign and transient. Therefore, the first step in assessment should be to consider the possibility of a false positive result, which can be caused by alkaline urine (pH >7), gross hematuria, mucus, semen or leukocytes.

Also, the proteinuria may be transient or persistent. Transient, mild proteinuria can be caused by recent strenuous exercise, standing for long periods (orthostatic proteinuria), pregnancy, urinary tract infection (UTI) and acute febrile illness. Congestive heart failure is a more serious cause of proteinuria that can also be transient. Orthostatic proteinuria is typically absent in the morning, occurs in the afternoon and is seen mainly in young adults.

In this case, there is mild proteinuria that could be transient or persistent. Therefore, the next best step would be to repeat the dipstick urine test early in the morning before the patient undergoes strenuous physical activity or stands for long periods to eliminate some of the contributing factors to transient proteinuria.

Transient proteinuria can be confirmed by a repeat dipstick result which is negative, in the absence of any suspected transient cause. Persistent proteinuria can be confirmed by two or more consecutive positive dipsticks over a one- to two-week period.

If the proteinuria turns out to be persistent, urine microscopy (and culture and sensitivity) (option E), kidney function tests including urea, creatinine, and electrolytes (option A) must be considered
as well as quantification of the urinary protein using urinary albumin to creatine ratio (ACR) (more convenient) (option D) or 24-hour urinary protein (gold standard) (option B).

34
Q

A 25-year-old man presents with a painless right scrotal lump which he noticed while taking a shower three days ago. There is no tenderness or other abnormal findings expect asymptomatic enlargement of the right testis. The epididymis and spermatic cord can be felt in their normal positions. An ultrasound scan is performed which is shown in the accompanying photograph.

Which one of the following is the most appropriate next step in management?

A. Reassurance.
B. Measurement of serum alpha phetoprotein (AFP).
C. Measurement of serum carcinoembryonic antigen (CEA).
D. Trans-inguinal excision of the right testis.
E. Trans-scrotal excision of the right testis.

A

Correct Answer Is B.

The photograph pictures the ultrasonographic view of the right testis, in which a lesion is seen. The lesion is combination of both cystic and solid structures. A testicular lump in a young man is testicular cancer until proven otherwise.

When testicular cancer is suspected on ultrasound, the next best step is always obtaining blood sample for serum tumor markers. Three serum tumor markers have established roles in patients with suspected testicular cancer: alpha fetoprotein (AFP), beta subunit of human chorionic gonadotropin (beta-hCG), and lactate dehydrogenase (LDH).

Tumor markers differ by histological subtype of testicular malignancies. In pure seminoma testicular cancers, AFP is never elevated and beta-hCG is elevated in fewer than 20% of patients. In other histological subtypes, AFP and beta-hCG are elevated in 85% of patients.

Of the options only measurement of serum AFP can be correct as the next best step in management. However, the best option would recommend both AFP ans beta-hCG.

Other measures to conisder prior to surgery are:
* High resolution CT scan of the abdomen, pelvis and chest
* Chest X-ray
* Chemistry profile and full blood count (FBC)

(Option A) No man with a testicular lump can be reassured unless thorough investigations exclude
testicular malignancies.

(Option C) CEA is not a tumor marker for testicular cancer.

(Options D and E) Tumor marker studies and imaging studies should be performed prior to surgical
management. If surgery is planned a trans-inguinal approach should be considered, as there is a risk of tumor spread and scrotal seeding when the testis is reached through scrotum.

35
Q

Jared, a 65-year-old man presents to your practice with the complaint of recurrent left flank pain and dark-colored urine with the last incidence being three days ago. He describes the pain as colicky and 7 out of 10 in its peak intensity. He is otherwise healthy, takes no medications, denies any recent trauma, fever, rigor, or chills, and has stable vital signs on physical examination. A urine dipstick test is positive for blood but negative for WBC, nitrite, or protein. Which one of the following is likely to be of the highest diagnostic value for Jared?

A. Urine culture.
B. Abdominal CT scan.
C. KUB CT scan.
D. KUB ultrasound.
E. Cystoscopy.

A

Correct Answer Is C.

The two main symptoms Jared has presented with are flank pain and dark-colored urine which is most like gross hematuria based on a positive office urine test for blood. Malignancies, infections (pyelonephritis), and renal stones are the most important differential diagnosis for this patient.

While a formal urinalysis and culture should be considered for him to exclude infection, it is unlikely that a urine culture (option A) yields any diagnostic results given the lack of fever, chills, rigors, and negative infection parameters in the urine test (negative WBC and nitrite).

Jared requires imaging studies for assessment of the presence of any tumors, stones, or anatomical abnormalities, and a non-contrast CT scan of the kidneys, ureters, and bladder (KUB CT scan) is the standard and most appropriate non-invasive modality, especially for Jared as for him, urolithiasis (urinary tract stones) seems to be the most likely explanation. Some studies suggest that a KUB CT is 95% sensitive and 100% specific for nephrolithiasis.

Cystoscopy is the gold-standard study for painless hematuria to exclude bladder cancer. With the flank pain in the picture, the bladder is less likely to be the source of bleeding, and cystoscopy (option E) is unlikely to be of diagnostic value.

Ultrasonography of the kidneys, ureters, and bladder (KUB ultrasound) (option D) is an excellent alternative for a KUB CT scan in cases of renal stones, especially if combined with a KUB x-ray. The reliability is much less in cases such as malignancies.

A regular abdominal CT scan (option B) is neither as sensitive nor specific for the evaluation of
renal system pathologies.

36
Q

A 25-year-old patient is being staged for further management of a seminoma that presented as a lump in the left testis. Involvement of which one of the following lymph node groups by lymphatic spread would be associated with the worst prognosis?

A. Cervical.
B. Mediastinal.
C. Retroperitoneal.
D. Inguinal.
E. Femoral canal.

A

Correct Answer Is A.

Testicular neoplasms spread via draining lymphatics along the gonadal veins in the spermatic cord, and along the inguinal canal to retroperitoneal nodes and from retroperitoneal and para-aortic nodes, progressive spread can occur via the cistern chyli and thoracic duct to mediastinal and leftsided cervical nodes at the base of the neck. The latter group drain into the systemic venous circulation; therefore, involvement of cervical nodes is associated with the worst prognosis.

Inguinal and femoral vessels and nodes (Cloquet) drain the leg and the skin of lower abdomen, back, buttock, penis, scrotum and anal canal. They do not drain testes.

37
Q

A 47-year-old man presents to your practice with a scrotal swelling. The swelling is not painful and has been there for the past 4 months. He does not feel bothered with the swelling and has just come to see you because her wife has forced him to. On examination, there is a non-tender swelling in the left testis. Ultrasonography shows the mass to be cyst within the right left testis. Which one of the following would be the next best step in management?

A. Surgery.
B. Biopsy through inguinal route.
C. Biopsy through scrotum.
D. Measuring serum tumour makers (alpha-phetoprotein and beta-hCG).
E. Reassurance.

A

Correct Answer Is D.

Testicular cystic lesions are relatively common findings on testicular ultrasound. The underlying pathology ranges from benign to malignant and may include:
* Simple testicular cyst
* Tunica albuginea cyst
* Cystic transformation of rete testis (intratesticular tubular ectasia)
* Intratesticular spermatocele
* Intratesticular varicocele
* Testicular epidermoid cyst
* Cystadenoma of the rete testis
* Testicular abscess
* Testicular tumors

Over 23% of testicular cysts are malignant. For this reason, malignancy should be excluded by serum tumor markers as the most appropriate next step in management once a testicular cyst is found on ultrasound. Three serum tumor markers have established role in testicular cancer:
1. alpha fetoprotein (AFP)
2. beta subunit of human chorionic gonadotropin (beta-hCG)
3. lactate dehydrogenase (LDH).

In seminoma testicular cancer, AFP is never elevated and beta-hCG is elevated in fewer than 20% of patients. In non-seminoma testicular cancers, AFP and beta-hCG are elevated in 85% of patients.

(Option A) Surgery for orchiectomy is considered after investigations establish a diagnosis of testicular cancer. Without a definite diagnosis, surgery is not indicated for now.

(Option B) Biopsy through inguinal approach is inappropriate at this stage and before tumor marker assay.

(Option C) Testicular tumors are never resected through scrotal excisions.

(Option E) This patient cannot be reassured unless full assessment excludes malignancy.

38
Q

A 17-year-old boy presents to the emergency department with a 2-hour history of a painful and tender right testis. On examination, the scrotum is tender and swollen. Spermatic cord is thickened and can be moved above the lump. Which one of the following is the next best step in management?

A. Surgery.
B. Ultrasound.
C. Urine microscopy and culture.
D. Full blood count.
E. Technetium scan.

A

Correct Answer Is A.

A painful scrotum should be taken seriously because the outcome of an untreated testicular torsion can be catastrophic. The testis can be infarcted in 4 to 6 hours.

With physical findings of a swollen, tender and firm hemiscrotum with a short and thickened spermatic cord the patient should immediately be taken to the operating room. No male younger than 18 years should be considered to have epididymo-orchitis until the testis has been exposed at operating room and torsion is excluded.

(Option B) The role of ultrasound in distinguishing between testicular torsion and epididymoorchitis is controversial, as it cannot reliably detect changes that are diagnostic for an early torsion.

(Options C and D) Once torsion of testis has been ruled out, full blood count, urine microscopy and Chlamydia testing are performed to clinch other possible diagnoses.

(Option E) A technetium-99 m scan can differentiate between the two conditions. In testicular torsion the testis is avascular, while it is hyperemic in epididymo-orchitis. However, with acute scrotal pain in a young male, the diagnosis is testicular torsion until proven otherwise by surgical exploration. No time should be wasted on diagnostic procedures.

39
Q

A 20-year-old man patient presents to your clinic with painless scrotal swelling. Ultrasound shows a solid mass in the left testis. Which one of the following is the next best step in management?

A. Fine needle aspiration cytology (FANC).
B. Surgical exploration.
C. CT scan of the chest.
D. Serum lactate dehydrogenase (LDH).
E. Serum beta-hCG and alpha fetoprotein (AFP).

A

Correct Answer Is E.

All scrotal lumps in a young male is malignancy until proven otherwise, especially if painless.

Ultrasound is the initial investigation of choice for assessment of a scrotal lump. If a lesion within the testis is found on ultrasound, malignancy should come at the top of the differentials list, and blood sample be obtained for serum tumor markers. Tumor markers used for testicular cancer are alpha feto-protein (AFP) , beta hCG and lactate dehydrogenase (LDH). LDH is usually elevated in metastatic tumors.

In seminoma testicular cancers, AFP is never elevated and beta-hCG is elevated in less than 20% of patients. In non-seminoma testicular cancer, however, AFP and beta hCG are elevated in 85% of patients. Serum tumor markers, especially AFP and beta hCG, can provide evidence for the initial diagnosis of a testicular cancer, and are useful for assessment of the prognosis and risk stratification.

(Option A) Fine needle aspiration is not recommended for testicular cancers due to potential risk of tumor implantation in the scrotal wall.

(Option B) Surgical exploration is contraindicated due to the risk of tumor spread. Any scrotal incision should be avoided. Once the diagnosis of testicular cancer is established, orchiectomy through inguinal approach is the best initial treatment.

(Option C) CT scan of the chest is helpful for tumor staging once the diagnosis is established.

(Option D) LDH levels raise when the tumor has metastasized and is not elevated early in the course of the disease when no metastasis has occurred; therefore, this marker alone is not often diagnostic.

40
Q

A 42-year-old man presents to your clinic with history of a slowly-enlarging painless scrotal mass over the past m onths. He has previous history of unilateral cryptorchidism which required orchiopexy. On examination, a solid scrotal lump is palpated. Which one of the following would be the next best step in management?

A. Measuring serum alpha fetoprotein.
B. Measuring serum human beta chorionic gonadotropin (βHCG).
C. CT scan of the abdomen and pelvis.
D. Ultrasonography of the scrotum.
E. Scrotal needle biopsy.

A

Correct Answer Is D.

All solid scrotal lumps should be considered malignant until proven otherwise. In approach to scrotal masses the most appropriate next step is to request an ultrasound to further evaluate the
scrotal mass.

Ultrasound of the testes can reliably diagnose the testicular tumor with considerable precision and can also detect any invasion of the tunica albuginea. This man also has undergone orchipexy that is a risk factor for development of testicular cancer.

Risk factors of development of testicular cancer include personal or family history of any of the following:
* Cryptorchidism (undescended testes)
* Orchiopexy
* Testicular atrophy
* Previous testicular cancer

(Options A and B) Tumor markers (alpha fetoprotein and beta HCG) are not requested unless ultrasound suggests cancer.

(Option C) CT scan of the abdomen, pelvis and chest is done for staging purposes and only considered after confirmed diagnosis of testicular malignancy.

(Option E) Needle scrotal biopsy should be avoided because of potential risk of tumor implanting malignant cells in the scrotal wall.